Stay ahead of learning milestones! Enroll in a class over the summer!

G
Topic
First Poster
Last Poster
No topics here!
IMO Shortlist 2011, Algebra 7
orl   23
N Apr 15, 2025 by bin_sherlo
Source: IMO Shortlist 2011, Algebra 7
Let $a,b$ and $c$ be positive real numbers satisfying $\min(a+b,b+c,c+a) > \sqrt{2}$ and $a^2+b^2+c^2=3.$ Prove that

\[\frac{a}{(b+c-a)^2} + \frac{b}{(c+a-b)^2} + \frac{c}{(a+b-c)^2} \geq \frac{3}{(abc)^2}.\]

Proposed by Titu Andreescu, Saudi Arabia
23 replies
orl
Jul 11, 2012
bin_sherlo
Apr 15, 2025
IMO Shortlist 2011, Algebra 7
G H J
G H BBookmark kLocked kLocked NReply
Source: IMO Shortlist 2011, Algebra 7
The post below has been deleted. Click to close.
This post has been deleted. Click here to see post.
orl
3647 posts
#1 • 12 Y
Y by tastymath75025, tritanngo99, Adventure10, Mango247, Rounak_iitr, ehuseyinyigit, cubres, and 5 other users
Let $a,b$ and $c$ be positive real numbers satisfying $\min(a+b,b+c,c+a) > \sqrt{2}$ and $a^2+b^2+c^2=3.$ Prove that

\[\frac{a}{(b+c-a)^2} + \frac{b}{(c+a-b)^2} + \frac{c}{(a+b-c)^2} \geq \frac{3}{(abc)^2}.\]

Proposed by Titu Andreescu, Saudi Arabia
This post has been edited 2 times. Last edited by orl, Jul 24, 2012, 1:35 PM
Z K Y
The post below has been deleted. Click to close.
This post has been deleted. Click here to see post.
oneplusone
1459 posts
#2 • 35 Y
Y by Nguyenhuyhoang, huyvietnam, sayantanchakraborty, MathPanda1, AdBondEvent, quangminhltv99, JasperL, AlgebraFC, yiwen, Wizard_32, abdelkrim, HolyMath, karitoshi, Wizard0001, srijonrick, megarnie, myh2910, ThisNameIsNotAvailable, qwedsazxc, WinterSecret, Adventure10, Mango247, ehuseyinyigit, Rayanelba, farhad.fritl, and 10 other users
It is easy to check that $a+b-c>0$ and its cyclic counterparts. Then by Holder's,
\[\sum_{cyc}{\frac{a}{(b+c-a)^2}}\sum_{cyc}a^2(b+c-a)\sum_{cyc}a^3(b+c-a)\geq (\sum_{cyc}a^2)^3=27\]
By Schurs,
\[\sum_{cyc}a^2(b+c-a)\leq 3abc
\] and
\[\sum_{cyc}a^3(b+c-a)\leq abc(a+b+c)
\] Therefore
\[\sum_{cyc}{\frac{a}{(b+c-a)^2}}\geq \frac{9}{(abc)^2(a+b+c)}\geq\frac{3}{(abc)^2}\]
Z K Y
The post below has been deleted. Click to close.
This post has been deleted. Click here to see post.
daniel73
253 posts
#3 • 7 Y
Y by yshk, Adventure10, Mango247, and 4 other users
For those who do not quickly see why $b+c-a>0$:

Click to reveal hidden text

Alternative solution without Holder's inequality:

Click to reveal hidden text
Z K Y
The post below has been deleted. Click to close.
This post has been deleted. Click here to see post.
mathbuzz
803 posts
#4 • 4 Y
Y by ehuseyinyigit, Adventure10, Mango247, and 1 other user
for solving it , we first prove a claim
claim--- let x,y,z be 3 positive reals. they satisfy $min(x+y,y+z,z+x)> 2^{1/2}$ and $x^2+y^2+z^2=3$.
then x,y,z are the sides of a triangle.
proof--- here we clearly assume an ordering $x >= y >= z$.
then clearly min(x+y,y+z,z+x)=y+z. then y+z > $ 2^{1/2}$. then 2$(y^2+z^2)$ >= $(z+y)^2$ . so, $y^2+z^2$ >$1$. now , clearly , $y+z >x$ [if not , assume that $y+z <= x $. then $x >= y+z > 2^{1/2}$ . so, $x^2 >2$ , which contradicts $x^2+y^2+z^2$=3]. so , x,y,z are the sides of a triangle.

now , applying it to our problem , we get that , a,b,c are the sides of a triangle.so , put a=p+q , b=q+r , c=r+p.
then the given condition translates into the constraint
$2(p^2+q^2+r^2+pq+qr+rp)$=3 and then applying LM method is sufficient . [here we must keep in mind the condition min(p+q,q+r,r+p) >$ sqrt.2$ for choosing
the values of p,q,r for minimizing the expression (if required)]
. the minimum value is at p=q=r=1/2 [calculation is tedious!] :lol:

Click to reveal hidden text
This post has been edited 1 time. Last edited by mathbuzz, Aug 13, 2012, 12:15 PM
Z K Y
The post below has been deleted. Click to close.
This post has been deleted. Click here to see post.
mahanmath
1354 posts
#5 • 8 Y
Y by bvdsf, Adventure10, Mango247, and 5 other users
We had got this inequality at an exam :D , This is what I wrote that time :
Click to reveal hidden text
Z K Y
The post below has been deleted. Click to close.
This post has been deleted. Click here to see post.
Anzoteh
126 posts
#6 • 4 Y
Y by Adventure10, Mango247, and 2 other users
Here is mine. (Outline)
1. Show that $a, b, c$ are sides of triangle, so that $\exists x, y, z\in\mathbb R^{+}$ with $a=x+y, b=x+z, c=y+z$ (which is shown by someone else).
2. The inequality becomes $[x^{2}y^{2}(x+y)+y^{2}z^{2}(y+z)+z^{2}x^{2}(z+x)]\ge 12x^{2}y^{2}z^{2}$
3. Show that $x+y+z\le\frac{3}{2},$ and homogenize the expression in (2) by multiplying LHS by $\frac{2}{3} (x+y+z)$ and RHS by $\frac {1}{27} ((x+y)^{2}+(y+z)^{2}+(z+x)^{2})^{3}.$
4. Expand (3) and we only need to use Muirhead to settle everything (See below)!

So it would be: (all symmetric sum)
$(3\sum x^{8}y^{4}z^{0}+6\sum x^{8}y^{3}z^{1}+3\sum x^{8}y^{2}z^{2}+12\sum x^{7}y^{5}z^{0}+33\sum x^{7}y^{4}z^{1}+39\sum x^{7}y^{3}z^{2}+9\sum x^{6}y^{6}z^{0}+69\sum x^{6}y^{5}z^{1}+102\sum x^{6}y^{4}z^{2}+51\sum x^{6}y^{3}z^{3}+69\sum x^{5}y^{5}z^{2}+153\sum x^{5}y^{4}z^{3}+27\sum x^{4}y^{4}z^{4})\ge (8\sum x^{8}y^{2}z^{2}+48\sum x^{7}y^{3}z^{2}+96\sum x^{6}y^{4}z^{2}+72\sum x^{6}y^{3}z^{3}+56\sum x^{5}y^{5}z^{2}+240\sum x^{5}y^{4}z^{3}+56\sum x^{4}y^{4}z^{4})$
which is obvious by Muirhead

It would be incredible that an A7 problem can be solved by expanding alone.
Z K Y
The post below has been deleted. Click to close.
This post has been deleted. Click here to see post.
utkarshgupta
2280 posts
#7 • 2 Y
Y by Adventure10 and 1 other user
Here will
$\prod (a+b-c) \ge 1$ ?
Z K Y
The post below has been deleted. Click to close.
This post has been deleted. Click here to see post.
Abubakir
68 posts
#8 • 2 Y
Y by Adventure10, Mango247
mathbuzz wrote:
for solving it , we first prove a claim
claim--- let x,y,z be 3 positive reals. they satisfy $min(x+y,y+z,z+x)> 2^{1/2}$ and $x^2+y^2+z^2=3$.
then x,y,z are the sides of a triangle.
proof--- here we clearly assume an ordering $x >= y >= z$.
then clearly min(x+y,y+z,z+x)=y+z. then y+z > $ 2^{1/2}$. then 2$(y^2+z^2)$ >= $(z+y)^2$ . so, $y^2+z^2$ >$1$. now , clearly , $y+z >x$ [if not , assume that $y+z <= x $. then $x >= y+z > 2^{1/2}$ . so, $x^2 >2$ , which contradicts $x^2+y^2+z^2$=3]. so , x,y,z are the sides of a triangle.

now , applying it to our problem , we get that , a,b,c are the sides of a triangle.so , put a=p+q , b=q+r , c=r+p.
then the given condition translates into the constraint
$2(p^2+q^2+r^2+pq+qr+rp)$=3 and then applying LM method is sufficient . [here we must keep in mind the condition min(p+q,q+r,r+p) >$ sqrt.2$ for choosing
the values of p,q,r for minimizing the expression (if required)]
. the minimum value is at p=q=r=1/2 [calculation is tedious!] :lol:

Click to reveal hidden text

What does abbreviation "LM" mean?
Z K Y
The post below has been deleted. Click to close.
This post has been deleted. Click here to see post.
vjdjmathaddict
502 posts
#9 • 1 Y
Y by Adventure10
lagrange multiplier
Z K Y
The post below has been deleted. Click to close.
This post has been deleted. Click here to see post.
ahmedAbd
91 posts
#10 • 4 Y
Y by Wizard_32, kiyoras_2001, Adventure10, Mango247
What does Titu Andreescu have to do with Saudi Arabia?
Z K Y
The post below has been deleted. Click to close.
This post has been deleted. Click here to see post.
Medjl
757 posts
#11 • 1 Y
Y by Adventure10
ahmedAbd wrote:
What does Titu Andreescu have to do with Saudi Arabia?

he was invited to teach Saudian team
Z K Y
The post below has been deleted. Click to close.
This post has been deleted. Click here to see post.
mathcool2009
352 posts
#12 • 12 Y
Y by 62861, MathStudent2002, Makorn, niyu, sunfishho, aops29, yayups, myh2910, Adventure10, HamstPan38825, Mango247, Quidditch
I wasn't expecting this to be possible.
Degree 14 Polynomial Bash
Z K Y
The post below has been deleted. Click to close.
This post has been deleted. Click here to see post.
H.HAFEZI2000
328 posts
#13 • 3 Y
Y by 554183, Adventure10, Mango247
ahmedAbd wrote:
What does Titu Andreescu have to do with Saudi Arabia?

they probably paid for this
Z K Y
The post below has been deleted. Click to close.
This post has been deleted. Click here to see post.
Taha1381
816 posts
#14 • 1 Y
Y by Adventure10
oneplusone wrote:
It is easy to check that $a+b-c>0$ and its cyclic counterparts. Then by Holder's,
\[\sum_{cyc}{\frac{a}{(b+c-a)^2}}\sum_{cyc}a^2(b+c-a)\sum_{cyc}a^3(b+c-a)\geq (\sum_{cyc}a^2)^3=27\]

What was the motivation for this kind of Holder usage?
This post has been edited 2 times. Last edited by Taha1381, Oct 27, 2018, 2:38 PM
Z K Y
The post below has been deleted. Click to close.
This post has been deleted. Click here to see post.
WolfusA
1900 posts
#15 • 8 Y
Y by Sugiyem, PickleSauce, 508669, L567, mathscrazy, ehuseyinyigit, Adventure10, farhad.fritl
H.HAFEZI2000 wrote:
ahmedAbd wrote:
What does Titu Andreescu have to do with Saudi Arabia?
they probably paid for this
Ten oil barrels for sure.
Z K Y
The post below has been deleted. Click to close.
This post has been deleted. Click here to see post.
Math-wiz
6107 posts
#16 • 1 Y
Y by Adventure10
orl wrote:
Let $a,b$ and $c$ be positive real numbers satisfying $\min(a+b,b+c,c+a) > \sqrt{2}$ and $a^2+b^2+c^2=3.$ Prove that

\[\frac{a}{(b+c-a)^2} + \frac{b}{(c+a-b)^2} + \frac{c}{(a+b-c)^2} \geq \frac{3}{(abc)^2}.\]
Proposed by Titu Andreescu, Saudi Arabia

Everything is fine, but why Titu Andreescu, Saudi Arabia! Sorry if this is considered spam
This post has been edited 1 time. Last edited by Math-wiz, Nov 9, 2019, 4:50 AM
Z K Y
The post below has been deleted. Click to close.
This post has been deleted. Click here to see post.
solver1104
510 posts
#17 • 2 Y
Y by Adventure10, Mango247
H.HAFEZI2000 wrote:
ahmedAbd wrote:
What does Titu Andreescu have to do with Saudi Arabia?

they probably paid for this

did you not see this
Z K Y
The post below has been deleted. Click to close.
This post has been deleted. Click here to see post.
Math-wiz
6107 posts
#18 • 1 Y
Y by Adventure10
solver1104 wrote:
H.HAFEZI2000 wrote:
ahmedAbd wrote:
What does Titu Andreescu have to do with Saudi Arabia?

they probably paid for this

did you not see this

Ohh, sorry :)
Z K Y
The post below has been deleted. Click to close.
This post has been deleted. Click here to see post.
ZETA_in_olympiad
2211 posts
#20
Y by
Note that $a+b-c, b+c-a$ and $a+c-b$ are positive. Thus by Hölder: $$\sum_{\text{cyc}} a^3(b+c-a) \sum_{\text{cyc}} a^2(b+c-a) \sum_{\text{cyc}} \frac{a}{(b+c-a)^2} \geq 27.$$Since by Schur: $$3ab\geq \sum_{\text{cyc}} a^2(b+c-a) \quad \text{and} \quad abc(a+b+c)\geq \sum_{\text{cyc}} a^3(b+c-a).$$Thus the inequality of the problem holds.
Z K Y
The post below has been deleted. Click to close.
This post has been deleted. Click here to see post.
strong_boy
261 posts
#21 • 2 Y
Y by Mango247, Mango247
It is easy to see $\sum \frac{a}{(b+c-a)^2} = \sum \frac{a^6}{a^5(b+c-a)^2} $ . Now we can use $T-2$ lemma :

$$\sum \frac{a^6}{a^5(b+c-a)^2} = \sum \frac{a^6}{(a^{\frac{5}{2}}(b+c-a))^2} \geq \frac{27}{(\sum a^{\frac{5}{2}}(b+c-a))^2}$$
Now by schur it is easy to see :

$$\sum a^{\frac{5}{2}}(b+c-a))^2 \leq abc(\sum \sqrt{a})$$
Now we only need to prove :
$$\frac{27}{abc(\sum \sqrt{a})^2} \geq \frac{3}{(abc)^2}$$
Now we need to prove $\sum \sqrt{a} \leq \sqrt{3}$ .And it is easy by Holder .
Z K Y
The post below has been deleted. Click to close.
This post has been deleted. Click here to see post.
awesomeming327.
1700 posts
#22
Y by
Let $S$ denote the left hand side. We know that by Schur's Inequality,
\begin{align*} 
a^{1.5}(a-b)(a-c)+b^{1.5}(b-c)(b-a)+c^{1.5}(c-a)(c-b)\ge 0\\
a^{1.5}bc+b^{1.5}ca+c^{1.5}ab\ge a^{2.5}(b+c-a) + b^{2.5}(c+a-b)+c^{2.5}(a+b-c)
\end{align*}Since $(\sqrt{a}+\sqrt{b}+\sqrt{c})^4\le 27(a^2+b^2+c^2)=81$ by Power Mean Inequality, we have
\[3abc \ge a^{2.5}(b+c-a) + b^{2.5}(c+a-b)+c^{2.5}(a+b-c)\]By Holder's Inequality, $S(3abc)^2 \ge (a^2+b^2+c^2)^3=27$ so the desired holds.
Z K Y
The post below has been deleted. Click to close.
This post has been deleted. Click here to see post.
sqing
41825 posts
#23
Y by
Let $a,b$ and $c$ be positive real numbers satisfying $\min(a+b,b+c,c+a) > \sqrt{2}$ and $a^2+b^2+c^2=3.$ Prove or disprove
$$ \frac{a}{(b+c-a)^3} + \frac{b}{(c+a-b)^3} + \frac{c}{(a+b-c)^3} \geq \frac{3}{(abc)^3}$$
Z K Y
The post below has been deleted. Click to close.
This post has been deleted. Click here to see post.
S.Das93
708 posts
#24
Y by
Can we solve this in a similar way by showing the Triangle Inequality and then substituting the respective $a=u+v,b=v+w,c=w+u$ ?
Z K Y
The post below has been deleted. Click to close.
This post has been deleted. Click here to see post.
bin_sherlo
708 posts
#25
Y by
Since $2(a^2+b^2)\geq (a+b)^2>2$ we get $a+b>\sqrt 2>\sqrt{3-a^2-b^2}=c$ thus, $a,b,c$ satisfy the triangle inequality. Let $a=\frac{y+z}{2},b=\frac{x+z}{2},c=\frac{x+y}{2}$.
\[\sum{x}\sum{\frac{1}{x^2}}-\sum{\frac{1}{x}}=\sum{\frac{y+z}{x^2}}\overset{?}{\geq} \frac{384}{(y+z)^2(x+z)^2(x+y)^2}\]Let $x+y+z=3u, \ xy+yz+zx=3v^2,\ xyz=w^3$ where $u^2+v^2=2$.
\[3u.\frac{9v^4-6uw^3}{w^6}-\frac{3v^2}{w^3}\overset{?}{\geq} \frac{384}{(9uv^2-w^3)^2}\iff \frac{u(9v^4-6uw^3)}{w^6}\overset{?}{\geq} \frac{v^2}{w^3}+\frac{128}{(9uv^2-w^3)^2}\]Note that $v^4\geq uw^3$ holds and $u\geq 1$ since $2=u^2+v^2\leq 2u^2$.
\[\frac{u(9v^4-6uw^3)}{w^6}\geq \frac{3uv^4}{w^6}\geq \frac{3u^2}{w^3}\geq \frac{v^2}{w^3}+\frac{2u^2}{w^3}\geq \frac{v^2}{w^3}+\frac{2}{u^2v^4}\geq \frac{v^2}{w^3}+\frac{128}{(9uv^2-w^3)^2}\]As desired.$\blacksquare$
Z K Y
N Quick Reply
G
H
=
a